Integrating the Dirac Delta function

Click For Summary
The integration of the function involving the Dirac delta function is discussed, specifically the integral ∫(t-1)δ[(2/3)t - (3/2)]dt. The initial calculation suggests the result should be 5/4, derived from evaluating the delta function at t = 9/4. However, Mathematica provides a different answer of 15/8, prompting further investigation. Participants emphasize the importance of changing variables correctly, noting that the differential changes when substituting variables. Ultimately, accurate handling of the delta function and variable transformation is crucial for obtaining the correct result.
Bugeye
Messages
3
Reaction score
0

Homework Statement


I am trying to integrate the function
\int _{-\infty }^{\infty }(t-1)\delta\left[\frac{2}{3}t-\frac{3}{2}\right]dt

Homework Equations



The Attempt at a Solution


I think the answer should be \frac{5}{4} because \frac{2}{3}t-\frac{3}{2}=0 when t=9/4. then (9/4-1) = 5/4. However, when I put the equation into Mathematica, it gives me an anser of 15/8.
 
Physics news on Phys.org
Bugeye said:

Homework Statement


I am trying to integrate the function
\int _{-\infty }^{\infty }(t-1)\delta\left[\frac{2}{3}t-\frac{3}{2}\right]dt

Homework Equations



The Attempt at a Solution


I think the answer should be \frac{5}{4} because \frac{2}{3}t-\frac{3}{2}=0 when t=9/4. then (9/4-1) = 5/4. However, when I put the equation into Mathematica, it gives me an anser of 15/8.

There's more to it than that. Do a change of variables u=2t/3-3/2. Don't forget du isn't the same as dt.
 
Bugeye said:

Homework Statement


I am trying to integrate the function
\int _{-\infty }^{\infty }(t-1)\delta\left[\frac{2}{3}t-\frac{3}{2}\right]dt

Homework Equations



The Attempt at a Solution


I think the answer should be \frac{5}{4} because \frac{2}{3}t-\frac{3}{2}=0 when t=9/4. then (9/4-1) = 5/4. However, when I put the equation into Mathematica, it gives me an anser of 15/8.

You need to be really, really careful when dealing with things like δ(f(t)). In your case, just change variables to x = (2/3)t.
 
Dick said:
There's more to it than that. Do a change of variables u=2t/3-3/2. Don't forget du isn't the same as dt.

Great, I got it, thanks a lot.
 
Question: A clock's minute hand has length 4 and its hour hand has length 3. What is the distance between the tips at the moment when it is increasing most rapidly?(Putnam Exam Question) Answer: Making assumption that both the hands moves at constant angular velocities, the answer is ## \sqrt{7} .## But don't you think this assumption is somewhat doubtful and wrong?

Similar threads

  • · Replies 3 ·
Replies
3
Views
3K
  • · Replies 31 ·
2
Replies
31
Views
4K
Replies
7
Views
2K
Replies
1
Views
2K
Replies
8
Views
1K
  • · Replies 40 ·
2
Replies
40
Views
4K
  • · Replies 9 ·
Replies
9
Views
4K
  • · Replies 105 ·
4
Replies
105
Views
6K
  • · Replies 5 ·
Replies
5
Views
1K
  • · Replies 7 ·
Replies
7
Views
2K